精華區beta Math 關於我們 聯絡資訊
我想要問的是 non-measurable sets. 在Royden 4th Ed.中 有一題題目如下 Let E be a nonmeasurable set of finite outer measure. Show that there is a G-delta set G that contains E for which m*(E) = m*(G) while m*(G\E) > 0 在這裡m*是outer measure 的notation. 我知道怎麼建構一個G-delta set G 使得 m*(E) = m*(G), 但是 我不太瞭解要怎麼使 m*(G\E) > 0 我想說用反證法: 要是每一個滿足 m*(E) = m*(G) 的 G-delta set G (包含 E), 也都滿足 m*(G\E) <= 0, 那麼m*(G\E)=0. Then by outer approximation by G-delta sets, E is measurable. -><- 我好奇的是上面的論證對嗎? 我好困惑啊~~ -- ※ 發信站: 批踢踢實業坊(ptt.cc) ◆ From: 165.124.140.63
math1209 :yes. 就是這樣子. 04/20 12:19